Recent content by Jonothancroller

  1. J

    Generator help

    It seems we did hahahaha. At least I won't forget this :)
  2. J

    Generator help

    Ohhh okay, so in a generator, positive charges don't flow towards the negative terminal, but instead the positive terminal? Makes sense.
  3. J

    Generator help

    Thank you! You've been very helpful.
  4. J

    Generator help

    Do you reckon the answer is wrong then? Because with a commutator in a generator, it should just keep the polarities of X and Y the same constantly rather than changing their polarities, as if they were connect to a slip ring.
  5. J

    Generator help

    Wait wait I think I understand this one. So, an emf will still be induced in the circuit due to Faraday's law. Thus, when the voltmeter is placed in series with the circuit, it can detect the voltage of the circuit. However, the voltmeter being in circuit stops the other components (i.e. ammeter...
  6. J

    Generator help

    Okay cool, I get everything now apart from the last part. When we use the right-hand palm rule, we're considering positive charges rather than electrons (coz HSC likes the 1800's throwback) so would we instead get x to y, but positive charges, and so x is positive? When we use the left-hand palm...
  7. J

    Generator help

    Hmmm, I'm not sure how well I get that. Isn't Lenz's law only applicable for changes in magnetic fields? That's how I learnt it anyway. And then, if we do that right hand palm rule, we get our thumb pointing downwards, which I'm not really sure how it relates to current going anti-clockwise.
  8. J

    Generator help

    http://prntscr.com/h3czlr Final one. I don't get why it is B. Voltmeters don't work when placed in series? But ammeters and lamps should work, right? So why not C?
  9. J

    Generator help

    http://prntscr.com/h3cz83 And just for confirmation, it is D because there is no back emf in a generator right? Didn't know if they were just being tricky with wording.
  10. J

    Generator help

    https://prnt.sc/h3cyn8 Also, why C?
  11. J

    Generator help

    https://prnt.sc/h3cwy7 Can someone explain this? My thoughts were it would be D, as from the right hand rule we get that the current will move from X to Y. Hence, X will be the positive terminal and Y will be the negative terminal, as positives are attracted to negatives. However, am I wrong...
  12. J

    Australian Maths Competition

    Notice that the unshaded area must also be divided in half. By drawing a diagram with the centres of the circles and the perpendiculars from the centres to the dividing line, it can be shown that the line must pass through the midpoint between the centres of the spheres. Letting P be the origin...
  13. J

    Australian Maths Competition

    Hey, I've got some questions that I haven't been able to do. Would anyone care to do these questions as practice and offer an explanation? Thanks. http://imgur.com/a/YfigO
  14. J

    2009 Physics HSC Q7

    http://imgur.com/vm1I8cD Hey, so the answer is C but I really don't get how. Can someone show me the method to do this?
  15. J

    Motors and Generators Question

    Okay I think I understand that, thanks.
Top